Random process involving CDF and PDF of standard normal

Click For Summary
SUMMARY

The discussion focuses on deriving a random process \(X(t)\) that satisfies specific probabilistic conditions involving the standard normal cumulative distribution function \(\Phi(x)\) and probability density function \(\phi(x)\). The solution proposes using a zero-mean unit-variance Gaussian process, specifically \(X(t) = \frac{W(t)}{\sqrt{t}}\), where \(W(t)\) is a Wiener process. The calculations confirm that the covariance structure meets the independence requirements necessary for the validity of the probability expressions presented.

PREREQUISITES
  • Understanding of standard normal distribution functions: \(\Phi(x)\) and \(\phi(x)\)
  • Knowledge of Gaussian processes and their properties
  • Familiarity with Wiener processes (Brownian motion)
  • Basic concepts of probability theory, particularly joint distributions and independence
NEXT STEPS
  • Study the properties of Wiener processes and their applications in stochastic processes
  • Learn about Gaussian processes and their role in statistical modeling
  • Explore the implications of covariance structures in random processes
  • Investigate the relationship between independence and zero covariance in probability theory
USEFUL FOR

Mathematicians, statisticians, and data scientists interested in stochastic processes, particularly those working with Gaussian processes and their applications in probability theory.

JohanL
Messages
154
Reaction score
0

Homework Statement


Let
$$ \Phi(x)=\int_{-\infty}^{x} \frac{1} { \sqrt{2\pi} } e^{-y^2 /2} dy $$
and $$ \phi(x)=\Phi^\prime(x)=\frac{1} { \sqrt{2\pi} } e^{-x^2 /2} $$
be the standard normal (zero - mean and unit variance) cummulative probability distribution function and the standard normal probability density function, respectively. Find a random process $$(X(t), \, t \in \mathbb{R}) $$ such that the following calculation is valid:
\begin{align}P\left(X(1) ≤ 0, X(2) ≤ 0\right) &= P\left(X(1) ≤ 0, X(2)-\frac{1} {\sqrt{2}}X(1)≤-\frac{1} {\sqrt{2}}X(1)\right)\\&=\int_{-\infty}^{0} P\left(X(2)-\frac{1} {\sqrt{2}}X(1)≤-\frac{1} {\sqrt{2}}x\right)\phi(x) dx\\&=\int_{-\infty}^{0} \Phi(-x)\phi(x) dx =\int_{0}^{\infty} \Phi(x)\phi(x) dx =\Bigg[\frac{\Phi(x)^2} {2}\Bigg]^{\infty}_{0}=\frac{3} {8}\end{align}

Homework Equations

3. solution
A zero-mean unit-variance Gaussian process with
$$E(X(1)X(2)) = (\frac{1}{\sqrt 2})$$
, because for such a process
X(1) and $$X(2)− (\frac{1}{\sqrt 2}) X(1)$$ are independent
and
$$P(X(2)−(\frac{1}{\sqrt 2}) X(1) ≤ \frac{1}{\sqrt 2} x) = \Phi(−x)$$since $$X(2)− (\frac{1}{\sqrt 2}) X(1)$$ is a Gaussian random variable with variance 1/2.

If anyone could explain the sections (a) red, (b) green and (c) blue i would appreciate it.
 
Physics news on Phys.org
Have you learned Wiener processes yet? If you have then a description of a solution that is (to me) much simpler than the above is
$$X(t)=\frac{W(t)}{\sqrt{t}}$$
where ##W(t)## is a Wiener process (aka Brownian Motion).

By the way, I'm pretty sure that line 2 in the problem statement is wrong. It should be:

$$
\int_{-\infty}^{0} P\left(X(2)-\frac{1} {\sqrt{2}}X(1)≤-\frac{1} {\sqrt{2}}x\ \bigg\vert\ X(1)=x\right)\phi(x) dx\ \ \ \textbf{[2a]}
$$

otherwise the step from line (1) to (2) is not valid.
I think what they expect you to do to validate the red part, is to evaluate##E\left[X(1)\left(
X(2)− (\frac{1}{\sqrt 2}) X(1)
\right)\right]##. You should be able to show this is zero. However, it would be a mistake to think that means they are independent. Zero covariance is a necessary but not sufficient condition for independence.

So I suggest you forget their solution, which looks dodgy to me. Instead, consider the following:

The step from (1) to (2a) is true if ##X(1)\sim N(0,1)##. [Condition A]
The step from 2a to 3 is valid if ##X(2)− (\frac{1}{\sqrt 2}) X(1)\sim N(0,1)## and is independent of ##X(1)##. [Condition B]
All other steps are identities, and hold regardless of the distribution.

For a Wiener Process ##W(t)##, we have ##W(t)\sim N(0,t)## and ##W(t)-W(s)## is independent of ##W(s)## for ##t>s##. So conditions A and B can easily be shown to be met for the process ##X(t) =\frac{W(t)}{\sqrt{t}}##. Hence ##X(t)## is a solution.
 
  • Like
Likes JohanL
JohanL said:

Homework Statement


Let
$$ \Phi(x)=\int_{-\infty}^{x} \frac{1} { \sqrt{2\pi} } e^{-y^2 /2} dy $$
and $$ \phi(x)=\Phi^\prime(x)=\frac{1} { \sqrt{2\pi} } e^{-x^2 /2} $$
be the standard normal (zero - mean and unit variance) cummulative probability distribution function and the standard normal probability density function, respectively. Find a random process $$(X(t), \, t \in \mathbb{R}) $$ such that the following calculation is valid:
\begin{align}P\left(X(1) ≤ 0, X(2) ≤ 0\right) &= P\left(X(1) ≤ 0, X(2)-\frac{1} {\sqrt{2}}X(1)≤-\frac{1} {\sqrt{2}}X(1)\right)\\&=\int_{-\infty}^{0} P\left(X(2)-\frac{1} {\sqrt{2}}X(1)≤-\frac{1} {\sqrt{2}}x\right)\phi(x) dx\\&=\int_{-\infty}^{0} \Phi(-x)\phi(x) dx =\int_{0}^{\infty} \Phi(x)\phi(x) dx =\Bigg[\frac{\Phi(x)^2} {2}\Bigg]^{\infty}_{0}=\frac{3} {8}\end{align}

Homework Equations

3. solution
A zero-mean unit-variance Gaussian process with
$$E(X(1)X(2)) = (\frac{1}{\sqrt 2})$$
, because for such a process
X(1) and $$X(2)− (\frac{1}{\sqrt 2}) X(1)$$ are independent
and
$$P(X(2)−(\frac{1}{\sqrt 2}) X(1) ≤ \frac{1}{\sqrt 2} x) = \Phi(−x)$$since $$X(2)− (\frac{1}{\sqrt 2}) X(1)$$ is a Gaussian random variable with variance 1/2.

If anyone could explain the sections (a) red, (b) green and (c) blue i would appreciate it.

Letting ##X_1 = X(1)## and ##X_2 = X(2)##, the random variable ##Y = X_2 - a X_1 ## has mean 0 and variance
\text{Var}(Y) = \text{Var}(X_1) + a^2 \, \text{Var}(X_1) - 2 a \, \text{Cov}(X_1, X_2)
What do you get when ##\text{Var}(X_1) = \text{Var}(X_2) = 1, a = 1/\sqrt{2}## and ##\text{Cov}(X_1 X_2) =1/ \sqrt{2}\;##?

So, you want ##X_2 - X_1/\sqrt{2}## to be independent of ##X_1## and to have variance 1/2; that is, you want
X_2 = X_1/\sqrt{2} \pm Z/\sqrt{2},
where ##Z \sim \text{N}(0,1)## is independent of ##X_1##. Can you do this in terms of ##W(1)## and ##W(2)\:##? What sign in ##\pm## must you take in order to get the result in eq.(3)?
 
  • Like
Likes JohanL
Question: A clock's minute hand has length 4 and its hour hand has length 3. What is the distance between the tips at the moment when it is increasing most rapidly?(Putnam Exam Question) Answer: Making assumption that both the hands moves at constant angular velocities, the answer is ## \sqrt{7} .## But don't you think this assumption is somewhat doubtful and wrong?

Similar threads

  • · Replies 13 ·
Replies
13
Views
2K
Replies
4
Views
2K
  • · Replies 105 ·
4
Replies
105
Views
6K
  • · Replies 1 ·
Replies
1
Views
2K
  • · Replies 5 ·
Replies
5
Views
2K
Replies
4
Views
2K
Replies
3
Views
2K
  • · Replies 11 ·
Replies
11
Views
3K
  • · Replies 4 ·
Replies
4
Views
2K